If a student can type 120 if if if if a student can type 120 words in 3
minutes, at this rate, how many words can
she type in 5 minutes?
(C) 180
(A) 240
(D) 160
(B) 200
(E) 120

Answers

Answer 1

Answer:

200

Step-by-step explanation:

Given: they can type 120 words in 3 minutes

Find number of words they can type in 1 minute: Words typed in 1 minute = 120/3=40

Solution: Words typed in 5 minutes = 40 * 5 = 200

Answer 2

Answer:

200 words

Step-by-step explanation:

First we need to find the words per minuite the student can type

we can do this by taking the number of words typed in 3 mins and dividing it by 3

120/3 = 40

Now we know she types 40 words per min

multiply 40 by 5 to get the no. of words typed in 5 mins

40*5=200

200 is the words she types in 5 mins

pls give brainliest


Related Questions

What are the zeroes of f(x) = x^2 + 5x + 6? (4 points)

A) x = -2, -3
B) x = 2,3
C) x= -2,3
D) x = 2, -3

Answers

Answer:

the answer is A, if you need explanation comment

Step-by-step explanation:

The answer is a but you need an explanation

PLEASE PLEASE PLEASE HELP ME. IM STUCK IN A SUMMER SCHOOL AND IF I DONT GET GOOD GRADES ILL GET IN BIG TROUBLE AND IM TOTALLY LOST ON THIS PROBLEM. I ALREADY DID THE FIRST TWO BUT I DONT KNOW HOW TO DO THE LAST ONE. PLEASE HELP ME!!!!

Answers

Answer:

We know that in the box there are:

4 twix

3 kit-kat

Then the total number of candy in the box is:

4 +3 = 7

a)

Here we want to find the probability that we draw two twix.

All the candy has the same probability of being drawn from the box.

So, the probability of getting a twix in the first drawn, is equal to the quotient between the number of twix and the total number of candy in the box, this is:

p = 4/7

Now for the second draw, we do the same, but because we have already drawn one twix before, now the number of twix in the box is 3, and the total number of candy in the box is 6.

this time the probability is:

q = 3/6 = 1/2

The joint probability is the product of the individual probabilities, so here we have

P = p*q = (4/7)*(1/2) =  2/7

b) same reasoning than in the previous case:

For the first bar, the probability is:

p = 3/7

for the second bar, the probability is:

q = 2/6 = 1/3

The joint probability is:

P = p*q = (3/7)*(1/3) = 1/7

c) Suppose that first we draw a twix.

The probability we already know that is:

p = 4/7

Now we want another type, so we need to draw a kit-kat, the probability will be equal to the quotient between the remaining kit-kat bars (3) and the total number of candy in the box (6)

q = 3/6

The joint probability is:

P = p*q = (4/7)*(3/6) = 2/7

But, we also have the case where we first draw a kit-kat and after a twix, so we have a permutation of two, then the probability in this case is:

Probability = 2*P = 2*2/7 = 4/7

A cubical water tank can contain 1000/125 cubic meters of water. Find the length of a
side of the water tank.
2 meters 3 meters
1/2 meters
1/3 meters

Answers

Answer:

2 meters.

Step-by-step explanation:

We know that a cube of sidelength L has a volume:

V = L^3

Here, we know that the volume of water that the cube can hold is:

(1000/125) m^3

Then the volume of our cube is exactly that:

V = (1000/125) m^3

Then we have the equation:

L^3 = (1000/125) m^3

Which we can solve for L

L = ∛((1000/125) m^3 ) = (∛1000/∛125) m

Where we used that:

∛(a/b) = ∛a/∛b

Solving the cubic roots, we get:

L = (10/5) m = 2m

The length of the side of the water tank is 2 meters.

Exercise (SARI) 1. Ram is four years older than Shyam. The sum of their ages is 52 years, Find the present age of Shyam​

Answers

Step-by-step explanation:

ratio =4:1

total= 4+1 =5

1/5 × 52= 10.4 years

Answer:

Step-by-step explanation:

let age of Shyam=x

age of Ram=x+4

x+x+4=52

2x=52-4=48

2x=48

x=48/2=24

age of Shyam=24 years

age of Ram=24+4=28 years.

Helpppp me plzzzzzzxzz

Answers

Can you please take a better picture so I can see it better

does anyone know the answer to this question?

Answers

Answer:

(1,2)

Step-by-step explanation:

1. plug x into the equation:

[tex]2<\frac{2}{3}(1)+2[/tex]

2. simplify:

[tex]2<\frac{2}{3}+2[/tex]

[tex]2<\frac{2}{3}+\frac{6}{3}[/tex]

[tex]2<\frac{8}{3}[/tex]

since 8/3 is greather than 2, (1,2) is a solution


Solve each equation. Round to four decimal places.
e^x=8

Answers

Answer:

x ≈ 2.0794

Step-by-step explanation:

Given

[tex]e^{x}[/tex] = 8 ( take ln of both sides )

ln [tex]e^{x}[/tex] = ln 8

x lne = ln 8 [ lne = 1 ]

x = ln 8 ≈ 2.0794 ( to 4 dec. places )

Answer:

x ≈ 2.0794

Step-by-step explanation:

[tex] \small \sf \: e {}^{x} = 8[/tex]

Take the natural logarithm of both side of equation

[tex]\small \sf \: In ( e {}^{x} ) = In ( 8 )[/tex]

[tex] \small \sf \: Use \: In(e {}^{x} ) = x \: to \: simplify \: the \: expression.[/tex]

x = In( 8 )

Write the number in exponential form.

x = In(2³)

Use In(aⁿ) = n × In ( a ) to transform the expression.

x = 3In( 2 ) ≈ 2.07944

Round to four decimal places = 2.0794

Find the angle between u = (8,-3) and v = (-3,8). Round to the nearest tenth of a degree.

Answers

Mark Brainliest please

Answer is 90 degrees

Find the value of x. Write it as a decimal.

Answers

Answer:

68.5

Step-by-step explanation:

Arc ABE = 360 - 2x

Arc AE = 2x

half the difference between the two arcs is 43

43 = [tex]\frac{1}{2}[/tex] [360 -2x - (2x)]    

Fifth grade students from Garden City Elementary School left for a field trip at the time shown on the clock.(6:54am) They returned from the field trip at 10:24 p.M. How long were the students on this field trip?

Answers

Answer:

15 hour(s) and 30 minute(s).

Step-by-step explanation:

Answer:

krkrkrl4k3?8999kekek

Step-by-step explanation:

the UK. I have been a while. I have been a while. I have been a while. I have been a while 6e5 the UK. I have been a while. I have been a while. I have

. How do we have a look at the moment. The comments for the first time in the UK. I

How many orders of magnitude bigger is 23,000 than 56?

Answers

Orders of Magnitude are in terms of scientific notation.

23,000 written in scientific notation is 2.3 x 10^4, it's magnitude is 4 ( the number 10 is raised to)

56 in scientific notation would be 5.6 x 10^1, its magnitude is 1.

4 - 1 = 3

2300 is 3 order of magnitudes larger.

The answer is 3.

The diagram shows a prism …,

Answers

Answer:

See figure below.

Step-by-step explanation:

See the figure below.

Q14SIMPLIFY THE EXPRESSION 6ab of2adivided by12x12ab+14a-a

Answers

Answer:

25

Step-by-step explanation:

6ab of 2a ÷ 12 × 12ab + 14a - a

= 6ab * 2a ÷ 12 × 12ab + 14a - a

= 12a²b ÷ 144ab + 13a

= 12*a*a*b / 144*a*b + 13*a

= a/12 + 13*a

= 1/12 + 13

= 1/25

Which property was used to simplify the expression?
30+9+4- 40+
distributive property
commutative property
associative property
inverse property
HELP SMB PLEASE!!!

Answers

Answer:

Distributive property.

Step-by-step explanation:

.

Answer:

holis polis trolis bsuslsgddnnbz

Identify a horizontal or vertical stretch or compression of the function by observing the equation of the function .

Answers

Bdbxbcjncnxndnsnns Identify a horizontal or vertical stretch or compression of the function by observing the equation of the function .hdbdbebqb

convert 9.56 cm = ________________ M​

Answers

Answer:

[see below]

Step-by-step explanation:

There are 100 cm in a meter.

9.56/100 = 0.0956

Hope this helps you.

(PS: You can mark the other user as Branliest)

An angle measures 16.4° less than the measure of its supplementary angle. What is the measure of each angle?

Answers

Answer:

Step-by-step explanation:

Let one angle = A

Let the other angle = B

A + B = 180

A = B - 16.4                   Substitute for A in the top equation

B - 16.4 + B = 180         Combine the left

2B - 16.4 = 180             Add 16.4 to both sides

2B = 180 + 16.4

2B = 196.4                    Divide by 2

B = 196.4/2

B = 98.2

A = B - 16.4

A = 98.2 - 16.4

A = 81.8

Problem: Construct a triangle with interior angle measures of 60° and 60°. Let one of the side lengths be 10. What are the lengths of the other sides?

Answers

Answer:

The lengths of the other sides would be 10, as well.

Step-by-step explanation:

Since we already know two of the three angles, we can subtract to find the other angle. The interior angles of a triangle will always add up to 180°. We have 60° and 60°, giving us 120°. 180 - 120 is equal to 60°, so that's the missing angle. Because all of the angles are the same, this is an equilateral triangle, which means the sides also have to be the same length. If one of the sides is 10, all of the sides are 10.

The triangle with sides 10 , 10 and 10 and all angles at 60° is an equilateral triangle

What is an Equilateral Triangle?

An equilateral triangle is a triangle in which all three sides have the same length.

Let the triangle be ΔABC , and

∠A = ∠B = ∠C = 60° and AB = BC = CA

Given data ,

Let the triangle be represented as ΔABC

Let the side length of the triangle be AB = 10

Now , the measure of ∠ABC = 60°

And , the measure of ∠BAC = 60°

So , the total sum of all the angles in a triangle = 180°

And , the measure of ∠ACB = 180° - ( 60° + 60° ) = 60°

Therefore , all the angles of the triangle are 60°

So , it is an equilateral triangle

And , the measure of side lengths of the triangle is 10

Therefore , AB = BC = CA = 10

Hence , it is an equilateral triangle

To learn more about equilateral triangle click :

https://brainly.com/question/17824549

#SPJ3

1+1+1+5+1+1+4+5+10+1LL pop

Answers

Answer:

The answer should be 30. I don’t know if those letters are apart of it but it’s 30.

I NEED ANSWER ASAP WILL GIVE BRAINLIEST(WORTH 30 POINTS)

Which function is represented by the graph?

A. f(x) = -2|x|+ 1
B. f(x) = -1/2|x|+1 C. f(x) = -2|x + 1|
D. f(x) =-1/2|x-1|

Answers

Answer: b

Step-by-step explanation:

Which of the following options results in a graph that shows exponential growth? (2 points)

f(x) = 0.4(3)x
f(x) = 3(0.5)x
f(x) = 0.8(0.9)x
f(x) = 0.9(5)−x

Answers

Answer:

Step-by-step explanation:

The formula for exponential equations is

[tex]y=a(b)^x[/tex] where a is the initial value and b is the growth rate. If b is greater than 1, we have growth; if  0 < b < 1, then we have decay. The only choice that shows a b value greater than 1 is the first one where b = 3. (The last one is also a decay as it can be written as

[tex]f(x)=0.9(\frac{1}{5x})[/tex] which isn't even exponential!)

Use Lagrange multiplier method to find the maximum and minimum values of (, , ) = − 2 + 5 On the sphere 2 + 2 + 2 = 30

Answers

Answer:

Please find the complete question in the attached file.

Step-by-step explanation:

Calculation with Lagrange multipliers of its optimum restricted places,

[tex]f_x = 2\ \ \ \ \ \ \ \ \ \ \ \ \ g_x = 2x\\\\f_y = 1 \ \ \ \ \ \ \ \ \ \ \ \ \ g_y = 2y\\\\[/tex]

Set the multiplier formulas for Lagrange:

[tex]f_x = \lambda g_x \to 2 = \lambda 2x ............ (i)\\\\f-y = \lambda g_y \to 1 = \lambda 2y................ (ii)\\\\constraint: \\\\\to x^2 + y^2 = 5 .................... (iii)\\\\Taking \ (i)\ divides\ (ii), \ (assuming\ \lambda \neq 0)\\\\\frac{2}{1}=\frac{\lambda 2x}{\lambda 2y}=\frac{x}{y}\\\\\therefore \\\\2y = x\\\\Sub\ into\ (iii)\ to\ find\\\\4y^2 + y^2 = 5 \to y = \pm 1\\\\[/tex]

We get solutions (x,y)=(2,1) and (−2,−1) if combined with 2y = x. These are all the identical points we obtained in (c) and we know the f(2,1) is the max, whereas f(−2, −1) is a minimum.

PLS HELP(algebra 1)
solve -7 + 18(17h + 19)

Answers

Answer:

335+306h

Step-by-step explanation:

solve the brackets first by distributing 18 to 17h and 19

-7+306h+342

then group the like terms

-7+342+306h

335+306h

Use the drop-down menus to identify the values of the parabola.
Vertex = ____
Domain = {x| ____ }
Range = {y| y ≤ ____ }

Answers

Answer:

Im guessing because their no chart.

Step-by-step explanation:

The answer is.

Vertex: 0,4.

Domain: X is a real number.

Range: 4.

What type of health screening would this patient most likely receive?
Sue is a 45-year-old woman with a family history of breast cancer. Her healthcare professional will most likely recommend that she receive a .

Answers

Answer:

A mammogram is what she would receive

Step-by-step explanation:

Find the length of AC. Round to the nearest hundredth if necessary.

Answers

i belive the answer is c

Which graph represents the solution of x2 + 9y2 ≤ 81 and y2 + 2 < x? On a coordinate plane, an ellipse has center (0, 0) and goes through (3, 0), (0, negative 9), (negative 3, 0), and (0, 9). A parabola opens to the right and goes through (6, 2), has vertex (2, 0), and goes through (6, negative 2). Everything inside of the ellipse and outside of the parabola is shaded. On a coordinate plane, an ellipse has center (0, 0) and goes through (3, 0), (0, negative 9), (negative 3, 0), and (0, 9). A parabola opens to the right and goes through (6, 2), has vertex (2, 0), and goes through (6, negative 2). Everything inside of the ellipse and inside of the parabola is shaded. On a coordinate plane, an ellipse has center (0, 0) and goes through (9, 0), (0, negative 3), (negative 9, 0) and (0, 3). A parabola opens to the right and goes through (6, 2), has vertex (2, 0), and goes through (6, negative 2). Everything inside of the ellipse and inside of the parabola is shaded. On a coordinate plane, an ellipse has center (0, 0) and goes through (9, 0), (0, negative 3), (negative 9, 0) and (0, 3). A parabola opens to the right and goes through (6, 2), has vertex (2, 0), and goes through (6, negative 2). Everything inside of the ellipse and outside of the parabola is shaded.

Answers

9514 1404 393

Answer:

  C

Step-by-step explanation:

The ellipse y-intercepts are ±3, the x-intercepts are ±9, eliminating choices A and B. The parabola is shaded inside, eliminating choice D.

The correct choice is the third one.

Solve for x. Round to the nearest tenth of a degree, if necessary.

Answers

Answer:

x ≈ 52.1°

Step-by-step explanation:

Using the tangent ratio in the right triangle

tan x = [tex]\frac{opposite}{adjacent}[/tex] = [tex]\frac{KL}{LM}[/tex] = [tex]\frac{36}{28}[/tex] , then

x = [tex]tan^{-1}[/tex] ([tex]\frac{36}{28}[/tex] ) ≈ 52.1° () to the nearest tenth )

what is the area of this triangle​

Answers

Answer:

son Judy even eat and technology and yyry and then he will get your BMW

What is the value of the median ?
1. 8
2. 4
3. 6
4. -4

Answers

The value of the median is going to be 6
Other Questions
Which of these is a tool for creating mobile apps? Appy PieC#Apple PieC++ the length of a building is 720 in. Use dimensional analysis to convert this quantity to yards. 1. Una carga Q1 = + 12 C se coloca a una distancia r = 0.024 m desde una carga Q2 = + 16 C. a) Determina la magnitud de la fuerza electrosttica que acta sobre las dos cargas, Q1 y Q2. b) Es la fuerza la atraccin o repulsin? 2. Determina la intensidad del campo elctrico a una distancia radial de r = 48 mm desde una carga de Q = 24 C. 3. Una carga Q1 = 24 mC se coloca a una distancia r = 0.032 m desde una carga Q2 = - 12 C. a. Determina la cantidad de energa potencial elctrica que tiene la carga Q1. b. Determina el potencial elctrico en la posicin de Q2. square root of v-5=6[tex] \sqrt{x - 5 = 6}[/tex] PLEASE HELP!!!! Its urgent Geometry, please answer question ASAP 11. A surveyor at point S discovers that the angle between peaks A and B is 3 times as large as the anglebetween peaks B and C. The surveyor knows that ZASC is a right angle. Find mzASs and m2BSC. 7(8+a)=189 please help and give me step by step Anyone wanna help with this please ? skilled manpower produce more manpowerhope you will answer briefly Managers make assumptions in CVP analysis. These assumptions include:__________a) constant total fixed costs. constant total variable costs. b) constant fixed cost per unit. c) constant sales volume. d) constant variable cost per unit. e) constant selling price per unit. You decide to go on a 4 day backpacking trip. The first day you walk 8 miles at northeast, on the second day, you walk 4 miles at eastsouth, and on the third day you walk 3 miles at southwest. On the fourth day you need to head straight back to your car. How far do you have to walk, and in what direction As a new manager, you will be responsible _______ scheduling all of the full-time employees as well as keeping track of the payroll.Select one:A. forB. byC. withD. to Which graph shows a set of ordered pairs that represent a function? Someone do this pls Which system has a central government and local units that implement the central governments decisions?A. monarchical systemB. federal systemC. parliamentary systemD. unitary system 2 Tables. A 2-column table with 2 rows is titled Table A. Column 1 is labeled Squares with entries 5, 10. Column 2 is labeled Circles with entries 3, 6. A 2-column table with 2 rows is titled Table B. Column 1 is labeled Squares with entries 10, 20. Column 2 is labeled Circles with entries 3, 9. Which statement is true about the ratios of squares to circles in the tables? The ratios in Table A are greater than the ratios in Table B. The ratios in Table B are greater than the ratios in Table A. Only some of the ratios in Table A are greater than the ratios in Table B. The ratios in Table A are equal to the ratios in Table B. tip tuyn ca th hm s x[tex]x^{3} -3x^{2} +2[/tex] ti im M(2,-2) As a fire rages through a forest, a large boulder is heated quickly. This rapid change in temperature causes the outer layer of the boulder to crack. weathering or erosion Joule is a SI unit of power Measuring cylinder is used to measure the volume of a liquid Which of the following characteristics are shared by all living organisms